LSAT考试全真试题三SECTION2精选6篇

SAT 2023-11-30 点击:

以下是小编整理的LSAT考试全真试题三SECTION2精选6篇,仅供参考,大家一起来看看吧。

【篇1】LSAT考试全真试题三SECTION2

section ⅱ

time-35 minutes

26 questions

directions: the questions in this section are based on the reasoning contained in brief statements or passages. for some questions, more than one of the choices could conceivably answer the question. however, you are to choose the best answer, that is the response that most accurately and completely answers the question. you should not make assumptions that are by commonsense standards implausible, superfluous, or incompatible with the passage. after you have chosen the best answer, blacken the corresponding space on your answer sheet.

1.an office building at state university contains a concrete stairway, the carpet on which has become worn and frayed. despite numerous warnings by the occupational safety and health administration, the university has not replaced a burned-out light in the stairway. fred, a student, recently caught his heel in the torn carpet, tripped, and fell down the stairway. he was hospitalized for a severe concussion and other injuries. after his release from the hospital, he required much more medical attention and medication and had to withdraw from the university for a semester. he brings suit against the university.

which one of the following is the best ling of causal analysis for fred"s attorney to pursue in the personal injury case?

(a) the concrete steps, because they were hard, worsened fred"s injuries.

(b) the university is responsible for the condition of the carpet.

(c) the burned-out light constitutes negligence.

(d) the distance fred fell worsened his injuries.

(e) the occupational safety and health administration has no jurisdiction over the university.

2.research shows that exercise has a beneficial effect on health. after much testing with many different types of persons, it has been shown that, in most cases, exercise definitely helps to prevent illnesses caused by viruses. the common cold is caused by a virus. therefore._________

which one of the following is the best completion of the argument above?

(a) exercise alone will not prevent the common cold

(b) exercise is no more effective than antibiotics in preventing the common cold

(c) exercise is probably not effective in preventing colds caused by bacteria

(d) exercise helps the body to destroy invading viruses

(e) exercise may help to prevent the common cold

3.just a few years ago salmon could not survive in the oxygen-starved and polluted thames. nor could many other species. but now, after years of determined effort, the salmon have returned, and that is a sure sign that the river is pollution-free.

each of the following indicates a possible flaw in the reasoning in the passage above eexcept:

(a) the salmon that have returned may be of a strain that is unaffected by the pollutants.

(b) the pollution may have been reduced to a level at which the salmon can survive.

(c) oxygen starvation is often a consequence of pollution, and this may have killed the salmon.

(d) the salmon may have been killed by one particular pollutant, which has now been removed while others remain.

(e) there may still be pollution, but its nature may have changed to a form that saimon can tolerate.

4.the state"s licensing procedures for day-care providers are inadequate.used to leave my son with a wonderful woman who kept a group of neighborhood children, but she quit rather than put up with the licensing paperwork. and a friend of mine tells me that he cannot recommend the day-care center that he uses, even though it is licensed, because the care providers are not interested in doing anything beyond meeting the minimum requirments of health and safety.

the author of the passage argues by

(a) providing examples to support two opposing positions

(b) basing a conciusion on specific cases

(c) disputing evidence cited by those with an opposing point of view

(d) predicting personal experience from a general principle

(e) using a generalization based on observation to undermine a theoretical principle

5.all 250 of the city"s democratic ward leaders were polled about the number of registered republicans who became registered democrats during the year. from the reports of the 20 leaders who replied to the questionnaire, we know that altogether they received a total of 500 former republicans into the democratic party within the year. projecting from this sample, we conclude that at least 6,000 republicans became democrats during the year.

which one of the following statements, if true, most clearly undermines the conclusion?

(a) several years ago, 5,000 members of the republican party registered as democrats.

(b) the more than 90 percent of ward leaders not replying had no former republicans registering as democrats.

(c) because they were too busy, more than 90 percent of the ward leaders did not reply percent of the ward leaders did not reply

(d) a number of the newly registered democrats returned to the republican party at a later date.

(e) similar figures can be produced from past years for registrationchanges from the democratic party to the republican party

6.a linguist recently argued that all human languages must have a common origin because some concepts are universal; that is they appear in all languages. for example, all languages are capable of describing lightness and darkness.

which one of the following, if true would most senously weaken the argument?

a) the bernese language does not contain basic nouns like automobile and airplane.

(b) no one linguist could possibly speak all known languages.

(c) all speakers, regardless of their languages are confronted with similar stimuli like lightness and darkness.

(d) the similarity between human language and dolphin language has not been attributed to a common origin.

(e) some languages include concepts of which speakers of other languages are not even aware.

7.all psychiatrists are doctors. only psychiatrists were invited to the conference. all who were invited to the conference stayed at the hefford hote. therefore, only psychiatrists stayed at the hefford hotel.

which one of the following statements, if added as a premise to the argument, would make the conclusion valid?

(a) only those who were invited to the conference stayed at the hefford hotel.

(b) all who were invited to the conference were psychiatrists.

(c) all psychiatrists were invited to the conference.

(d) no one who was invited to the conference failed to stay at the hefford hotel.

(e) only doctors stayed at the hefford hotel.

question 8-9

a thing is “nauseous” if it makes one sick to the stomach; the unfortunate victim of this malaise is “nauseated” . the common misuse of “nauseous” can be illustrated with the following sentence: “when he sits too long, turns his head too abruptly, or walks any distance, he gets dizzy, loses balance, and becomes nauseous.” he doesn"t become nauseous unless he turns other people"s stomachs; he becomes nauseated. a person who is nauseated is no more nauseous than a person who has been poisoned is poisonous.

8.based on the passage above, which phrase does not provide a logical completion to the following sentence?

a person who is nauseated is no more nauseous than a person who has been_______.

(a) murdered is murderous

(b) corrupted is corruptible

(c) awed is awesome

(d) irritated is irritating

(e) scared is scary

9.the author"s approach in the passage includes all of the following except

(a) explaining how a word is misused

(b) drawing an analogy

(c) providing an example

(d) relying on a word"s ambiguous meaning

(e) defining key terms

10. therapy, whether physical or social, is a counterirritant that aids in that equilibrium of the physical organs that protects the central nervous system. whereas pleasure is a counterirritant (for example, sports, entertainment, and alcohol), comfort is the removal of irritants. both pleasure and comfort are strategies of equilibrium for the central nervous system.

it can be concluded from the statements above that the exile of a deviant member of a group can be

(a) therapeutic for the rest of the group

(b) irritating for the rest of the group

(c) pleasurable for the rest of the group

(d) comforting for the rest of the group

(e) counterirritating for the rest of the group

11. the situation has gotten worse. unless elizabeth goes i must go. but she will not to if i stay so we will both have to go.

what is the flaw in the reasoning above?

(a) the first claim, that the situation has gotten worse, is irrelevant.

(b) a possible course of action is overlooked

(c) it is assumed that staying is the same as not going.

(d) the conclusion refers to a possibility that is not explicitly referred to in any of the premises.

(e) insufficient information is given about the circumstances.

12. student x: i"m worried about failing the course.

student y: don"t worry. as the professor said, any student who fails to submit a term paper will fail the course. so just make sure that you submit a term paper, and you will not fail the course.

which one of the following exhibits the same logical flaw as that exhibited in student y"s remark?

(a) any restaurant that serves paella without saffron is not authentic so if the restaurant serves paella with turmeric instead of saffron, it is authentic.

(b) any native fishers who earn their livings by fishing the local rivers and lakes are worth hiring as guides. so a person who is a native fisher is worth hiring as a guide.

(c) anyone who can consistently bowl over 200 points per game should become a professional bowler. if you can consistently bowl over 200 points per game, you should become a professional bowler.

(d) any engineer who cannot solve the equation in a reasonable amount of time will not get a license. so if you are an engineer who can solve the equation in a reasonable amount of time, you will get a license.

(e) any cook who is in a hot kitchen will have the kitchen. if you are a cook in a kitchen that is hot, you will be forced to leave the kitchen.

13. trade protection is bad policy. take the case of the microcomputer industry. the united states government attempted to restore the computer chip market to united states manufacturers, who had ceased production in the face of an abundant supply of cheap chips from foreign manufacturers. under trade protection, it was expected that, as government-imposed quotas and excise taxes forced the price of foreign chips to rise, united states manufactures would reenter the market. they did, but at only slightly lower prices that the now-high prices of foreign firms. the lesson has been simple: trade protection means that united states manufactures gain while united states manufactures gain while united states consumers lose.

which one of the following is an assumption on which the author"s argument relies?

(a) it is unreasonable to expect that government-imposed quotas and excise taxes will reduce prices for united states consumers.

(b) united states manufacturers of computer chips are more concerned with high profits that are foreign manufacturers.

(c) the united states government"s primary purpose in trade protection is to restore markets and profits to united states manufacturers.

(d) with respect to trade protection, the microcomputer industry is representative of united states industry in general.

(e) the quality of the chips produced by united states manufacturers is better than the quality of the chips produced by foreign manufacturers.

questions 14-15

until now, this painting was believed to be a self portrait of the artist, but it probably is not. true, the heavy gold chain worn by the gentleman in the picture is similar to one presented to the artist by the queen of england after he had painted her portrait. but the sword hanging at the gentleman"s hip suggests that he was a knight, a rank higher than any the artist ever attained.

14. the argument above depends on assuming that the self-portrait would

(a) include objects that are important to the artist

(b) not exaggerate the rank achieved by the artist

(c) be similar to portraits of others whom the artist had painted

(d) be verifiable only through the examination of objects in the painting

(e) not portray the artist engaged in the activity of painting

15. which one of the following, if true, would most strengthen the author"s argument?

(a) historians cannot determine whether the artist was ever knighted.

(b) the artist painted only one self-portrait in his entire life.

(c) several gentlemen of the artist"s day wore heavy gold chains like the one in the painting.

(d) the gentleman in the painting resembles portraits of the artist painted by other artists.

(e) the artist was the only gentleman of his day who received a heavy gold chain from the queen of england.

16. colleges boast about the great sizes of their libraries. they quote figures in the hundreds of thousands of volumes, but how many books can a student read in four years-a thousand? what good, then, are all the rest?

which one of the following statements most accurately expresses the author"s main point?

(a) college librarians do not have anything about which they can boast.

(b) no student can read more than a thousand books in four years.

(c) it is silly to boast about a library"s size, regardless of whether the library is large or small.

(d) unused books are a waste of taxpayers" money.

(e) college libraries are unnecessarily large.

17. eli:according to many scientists the widespread production and use of chlorofluorocarbons (cfcs)-e.g., to provide coolant for air conditioners-inevitably leads to their escape into the atmosphere where they destroy the vital ozone layer. in my opinion, the continued use of cfcs by humans is like a harmful habit, which, if unchecked, could have disastrous effects on the user, namely self-destruction. the obvious and necessary cure, therefore, is a complete ban on cfc production and use.

mark: the cure you propose would kill, not save, the user. a ban on cfc production and use would cause the destruction of the air conditioning industry.

the source of mark, and eli"s dispute is their lack of agreement on which one of the following terms?

(a) cure

(b) user

(c) ban

(d) production

(e) destruction

18. if there is a decrease in the number of homeless families, then either the number of available jobs has increased or else the cost of renting or purchasing housing has decreased. if the cost of renting or purchasing housing has decreased, then the supply of housing must be greater than the demand.

assume that there is a decrease in the number of homeless families. according to the passage, which one of the following statements. cannot be true?

(a) the number of jobs has been decreasing.

(b) the cost of renting or purchasing housing has been decreasing, and housing has been decreasing, and housing supply exceeds demand.

(c) the number of jobs has been increasing, and the cost of renting or purchasing housing has been stable.

(d) the number of jobs has been decreasing, the cost of renting or purchasing housing has been decreasing, and housing demand exceeds supply.

(e) the number of jobs has been increasing, the cost of renting or purchasing housing has been increasing, and housing supply exceeds demand.

19. everyone knew that if the team"s star player were too badly injured to play in saturday"s game, the team would lose. since the team won, the star player must have played

which one of the following is most similar in logie to the argument above?

(a) red spots on the feet are a symptom of a disease called borosis. this patient has green spots on his feet, therefore, he does not have borosis.

(b) if the frost comes late, the farmer has a good crop. because the frost was late, the farmer must have a good crop.

(c) the instructor in a course ruled that if a student received an a on either the term paper or the final exam, then the student would receive an a for the course. anne received an a on the term paper and a c on the final exam. therefore, anne received an a for the course.

(d) people in driving school realized that if pete had another auto accident, he would be expelled from the driving school, pete was not expelled from the driving school. therefore, he must not have had another auto accident.

(e) if swenson pays a $100 fine, he will not be expelled from the country club. because swenson has not been expelled, he must have paid the fine.

20. an examination of corruption provides the basis for rejecting the view that an exact science of society can ever be constructed. as with all other social phenomena that involve deliberate secrecy, it is intrinsically impossible to measure corruption, and this is not merely due to the fact that social science has not yet reached its goal, achievable to be sure, of developing adequate quantifying techniques. if people were ready to answer question about their embezzlements and bribes, it would their embezzlements and bribes, it would mean that these practices had acquired the character of legitimate, taxable activities and had ceased to be corrupt. in other words, corruption must disappear if it is to be measurable.

which one of the following most accurately states a hidden assumption that the author must make in order to advance the argument above

(a) some people believe that an exact science of society can be constructed.

(b) the primary purpose of an exact science to quantify and measure phenomena

(c) an intrinsic characteristic of social phenomena that involve deliberate secrecy is that they cannot be measured.

(d) an exact science of social phenomena that involve deliberate secrecy cannot be constructed.

(e) an exact science can be constructed only when the phenomena it studies can be measured.

question 21-22

this country does not need any more restrictions on pollution. in fact, we should abolish those that are currently on the books. in spite of various restrictions, pollution levels have actually increased over the past 40 years. most importantly, dramatic statistics show that, as pollution levels have increased over this period, the general health of the population has improved rather than deteriorated.

21. which one of the following assertions, if true, provides the most effective challenge to the author"s conclusion?

(a) the general health of this country"s population has improved over the past 40 years primarily because of new advances in medicine and nutrition, not because of pollution.

(b) there are several countries in the would in which there have been no restrictions on which there have been no restrictions on pollution over the past 40 years, and pollution levels have actually increased.

(c) similar statistics show that, in several countries, the general health of the population has improved over the past 40 years, while pollution levels have actually decreased.

(d) pollution levels would have increased even more than they have and the general health of the population would not have improved as much as it has without the restrictions that have been on the books.

(e) in the period prior to the past 40 years, pollution restrictions were nonexistent in this country and the general health of the population improved at a far slower rate than it did during the past 40 years.

22. the author concludes that there should be no restrictions on pollution partly because

(a) they are no longer needed to improve the general health of the population

(b) they reduce the positive effect that pollution has on the general health of the population

(c) statistics show that as pollution restrictions have increased so have pollution levels

(d) pollution has not yet reached levels that are detrimental to the health of the general population

(e) they are ineffective in lowering pollution levels.

23. in metropolitan areas, almost 60 percent of all fires are set by children, while in rural areas about 40 percent are a psychological survey discovered that all children who play with fire believe that there will be no consequences if their parents catch them doing it.

which one of the following inferences can be most reliably drawn from the passage above?

(a) most children who believe there will no consequences if they are discovered playing with fire do play with fire.

(b) parents who discover their children playing with fire will prevent those children from playing with fire in the future.

(c) if parents have successfully instilled in their children the belief that there will be consequences if they are caught playing with fire, these children have not been among those playing with fire.

(d) children who play with fire attach no sense of right or wrong to this action.

(e) most children who do not play with fire believe there will be consequences if their parents discover them playing with fire.

24. somewhere, somehow, what was once a perfectly good rule by which to live was twisted into the false and sinister idea that “money is the root of all evil” to the contrary, the proper use of money provides us with the food, clothes, health care, and shelter that we all need to sustain our lives.

the author"s argument is logically flawed in that it

(a) uses examples that do not refute the generalization that all evil is rooted in money

(b) uses inappropriate examples to demonstrate the proper use of money

(c) ignores some of the evil things that money can buy

(d) fails to acknowledge that food, clothes, health care, and shelter can sometimes lead to evil

(e) fails to recognize that money can be sued for a lot of other good things besides sustenance

25. in a recent experiment, a high school english teacher interspersed real,commonly used proverbs with several nonsensical proverbial-sounding statements that he had made up. he then asked his students to evaluate all of the statements on the list. in general, the students found the bogus proverbs and the real proverbs to be equally full of wisdom and meaning,. the teacher concluded that proverbs attain their status as proverbs more through frequent usage than through their inherent wisdom.

which one of the following if true, would most effectively challenge the teacher"s conclusion?

(a) some proverbs are used more frequently than others.

(b) there were more real proverbs than bogus proverbs in the list of statements.

(c) there are stylistic differences between proverbial and proverbial-sounding statements.

(d) some students view a statement in one way and other students view the same statement in a very different way.

(e) the students selected as evaluators were too inexperienced to judge the wisdom of the statements.

26. it is commonly accepted that we should be concerned about our own physical health. the desire to take responsibility for all aspects of our physical condition, however, produces a number of negative consequences. by focusing exclusively on our physical health, we tend to ignore out mental health. therefore, although we can derive physical benefits from our preoccupation with physical health, we often do so at the expense of our mental health.

the author establishes her position in the passage by doing which one of the following?

(a) she defends her position and then extencls it into a second area.

(b) she reveals a contradiction in a position commonly held to be correct.

(c) she supports a commonly held point of view by providing additional evidence

(d) she first states her position and then qualifies it with a number of concessions

【篇2】LSAT考试全真试题三SECTION2

section ii

time—35 minutes

25 questions

directions: the questions in this section are based on the reasoning contained in brief statements or passages. for some questions. more than one of the choices could conceivably answer the question. however, you are to choose the best answer, that is, the response that most accurately and completely answers the questions. you should not make assumptions that are by blacken the corresponding space on your answer sheet.

1. when politicians resort to personal atacks many editortalists criticize thest attacks but most voters pay them scant attention. eeveryone knows such attacks will end after election day, and politicians can be excused for mudslinging. political commentators, however, cannot be. political commentators should be engaged in sustained and senous debate about ideas and policies. in such a context personal attacks on opponents serve not to beat those opponents but to cut off the debate.

which of the following most accurately states the main point of the argument?

(a) dersonal attacks on opponets serve a usuful purpose for politicians.

(b) political commentators should not resort to personal attacks on their opponents.

(c) editonalists are right to criticize politicians who resort to personal attacks on their opponents.

(d) the purpose of serious debate about ideas and policies is to counteract the effect of personal attacks by politicians.

(e) voters should be concerned about the personal attacks politicians make on each other.

2. throughout the popoya islands community pressure is exerted on people who win the national lottery to share their good fortune with their neighbors. when people living in rural areas win the lottery they invariably throw elaborate neighborhood feasts, often wiping, out all of their lottery winmmings. however, in the cities, lottery winners frequently use their winnings for their own personal investment rather than sharing their good fortune with their neighbors.

which one of the following true, contributes most to an explanation of the difference between the behavior of lottery winners in rural areas and those in cities?

(a) twice as many popoyans live in rural areas as live in the city.

(b) popoyan city dwellers tend to buy several lottery tickets at a time, but they buy tickets less frequently than do rural dwellers.

(c) lottery winners in rural areas are notified of winning by public posting of lists of winners, but notification in the city is by private mail.

(d) families in rural areas in the popoyas may contain twelve or foruteen people, but city families average six or seven.

(e) twice as many lottery tickets are sold in rural areas as are sold in the city.

3. a new medication for migraine seems effective, but there is concern that the medication might exacerbate heart disease. if patiens with heart disease take the medication under careful medical supervision. however, harmful side effects can definitely be averted. the concern about those side effects is thus unfounded.

the argument depends on which one of the following assumptions?

(a) the new medication actually is effective when taken by patients with heart disease.

(b) no migraine sufferers with heart disease will take the new medication except under careful medical supervision.

(c) most migraine sufferers who have taken the new medication in trials also had heart disease

(d) the new medication has various other side effects, but none as serious as that of exacerbating heart disease.

(e) the new medication will displace all migrame medicztions currently being used.

4. the highest-ranking detectives in the city s police department are also the most adept at solving crimes. yet in each of the past ten years. the average success rate for the city s highest-ranking detectives in solving crimnal cases has been no higher than the average success rate for its lowest-ranking detectives.

which one of the follwing, if true, most helps to resolve the apparent paradox?

(a) the detectives who have the highest success rate in solving criminal cases are those who have worked as detectives the longest.

(b) it generally takes at least ten years for a detective to rise from the lowest to the highest ranks of the city s detective force.

(c) those detectives in the police department who are the most adept at solving criminal cases are also those most likely to remain in the police department.

(d) the police department generally gives the criminal cases that it expects to be the easiest to solve to its lowest-ranking detectives.

(e) none of the lowest-ranking detectivesin the police department had experiecne in solving critninal cases prior to joining the police deparment.

5. imgation runoff from neighboring farms may well have increased the concentration of phosphorus in the local swamp above previous levels, but the claim that the increase in phosphorus is harming the swamp s native aquatie wildlife is false: the phospborus concentration in the swamp is actually less than that found in certain kinds of bottled water that some people drink every day.

the argument is vulnerable to criticism on the ground that it

(a) makes exaggerations in formulating the claim against which it argues

(b) bases its conclusion on two contradictiry claims

(c) relies on evidence the relevance of which has not been established

(d) concedes the very point that it argues against

(e) makes a generalization that is unwarranted because the sources of the data on which it is based have not been specified.

6. copyright laws protect the rights of writers to profits earned from their writings. whereas patent laws protec: inventors rights to profits earned from their inventions in jawade, when computer-software writers demanded that their rights to profit be protected, the courts determined that information written for a machine does not fit into either the copyright or the patent category. clearly, therefore, the profit rights of computer-software writers remain unprotected in jawade.

which one of the following is an assumption on which the argument depends?

(a) computer-software writers are not an influential enough group in jawade for the government to consider modifying existing copyright laws in order to protect this group s profit rights.

(b) no laws exist, other than copyright laws and patent laws, that would protect the profit rights of computer-software writers in jawade.

(c) most of the computer software used in jawade is imported from other countries.

(d) computer software is more similar to writings covered by copyright laws than it is to inventions covered by patent laws.

(e) copyright laws and patent laws in jawade have not been modified since their original adoption.

7. brownlea s post office must be replaced with a larger one. the present one cannot be expanded. land near the present location in the center of town is more expensive than land on the outskirts of town. since the cost of acquiring a site is a significant part of the total construction cost, the post office clearly could be built more cheaply on the outskirts of town.

which one of the following, if true, most seriously undermines the argument s stated conclusion?

(a) the new post office will have to be built in accordance with a demanding new citywide building code.

(b) if the new post office is built on the outskirts of town, it will require a parking lot, but if sited near the present post office it will not.

(c) if the new post office is built on the outskirts of town, current city bus routes will have to be expanded to provide access.

(d) if the new post office is built on the outskirts of town, residents will make decreased use of post office boxes, with the result that mail carriers will have to deliver more mail to homes.

(e) if the new post office is built near the center of town, disruptions to city traffic would have to be minimized by taking such steps as doing some construction work in stages at night and on weekends.

8. in the past, the railroads in ostronia were run as regional monopelies and opeerated with little regard for what customers wanted. in recent years, with improvements to the ostronian national highway network the railroad companies have faced heavy competition from longdistance trucking companies. but because of government subsidies that have permitted ostronian railroad companies to operate even while incuring substantial losses, the companies continie to disregard customers needs and desires.

if the statements above are true, which one of the following must also be true on the basis of them?

(a) if the government of ostronia ceases to subsidize railroad companies. few of those companies will continue to operate.

(b) few companies in ostronia that have received subsidies from the government have taken the needs and desires of their customers into account.

(c) without government subsidies, railroad companies in ostronia would have to increase the prices they charge their customers.

(d) the transportation system in ostronia is no more efficient today than it was in the past.

(e) in recent years, some companies in ostronia that have had little regard for the desires of their customers have nonetheless survived.

9. although damon had ample time carlier in the month to complete the paper he is scheduled to present at a professional conference tomorrow morning he repeatedly put off doing it. damon could still get the paper ready in time, but only if he works on it all evening without interruption. however, his seven-year-old daughter s tap-dance recital takes place this evening and damon had promised both to attend and to take his daughter and her friends out for ice cream afterward. thus, because of his procrastination. damon will be forced to choose between his professional and his farmily responsibilities.

the argument proceeds by

(a) providing evidence that one event will occur in order to establish that an altemative event cannot occur

(b) showing that two situations are similar in order to justify the claim that someone with certain responsibilities in the first situation has similar responsibilities in the second situation

(c) invoking sympathy for someone who finds himself in a dilemma in order to excuse that person s failure to meet all of his responsibilities

(d) making clear the extent to which someone s actions resulted in harm to others in order to support the claim that those actions were irresponsible

(e) demonstrating that two situations cannot both occur by showing that something necessary for one of those situations is incompatible with something necessary for the other situation

10. the increase in the price of jet fuel is due to a sharp decrease over the past year in the supply of jet fuel available relative to demand. nonetheless, the amount of jet fuel available for sale is larger today than it was last year.

if the statements above are true, which one of the following conclusions can be properly drawn on the basis of them?

(a) the demand for jet fuel has increased over the past year.

(b) the fuel efficiency of jet engines has increased over the past year.

(c) the number of jet airline flights has decreased over the past year.

(d) the cost of refining petroleum for jet fuel has increased over the past year.

(e) the supply of petroleum available for jet fuel ha decreased over the past year.

questions 11-12

nan government subsidies have been proposed in cariana to encourage farmers in rochelle, the country s principal agricultural region, to implement certain new farming techniques unless these techniques are implemented erosion of productive topsoil cannot be controlled unfortunately farmers cannot afford to shoulder the entire cost of the new techniques, which are more expensive than those currently used therefore, without subsidies agricultural output in rochelle will inevitably decline

betty but erosion in rochelle is caused by recurring floods, which will end next year once cariana completes the hydroelectric dam it is building across the region s major river therefore, rochelle s total agricultural output will stabilize at its present level even without subsidres.

11. which one of the following is an assumption on which betty s argument depends?

(a) building a dam across rochelle s major river will not reduce any recurrent flooding that occurs in regions of cariana other than rochelle.

(b) the new farming techniques that must be implemented to control soil erosion in rochelle are not well suited to other regions of cariana

(c) the current yearly output, if any from rochelle s land that will be permanently under water once the dam is completed will at least be matched by additional yearly output from rochelle s remaining land

(d) the cost to the government of cariana to operate the hydroelectric dam will not be greater than the projected cost of subsidizing the farmers of rochelle in the implementation of the new farming techniques

(e) the government of cariana has sufficient financial resources both to subsidize its farmers implementation of new farming techniques and to operate a hydroelectric dam.

12. betty uses which one of the following argumentative techniques in countering alan s argument?

(a) showing that one premise in alan s argument is inconsistent with another premise in his argument

(b) making additional claims that, if correct undermine a premise in alan s argument

(c) demonstrating that alan s conclusion is true but not for the reasons alan gives to support it

(d) presenting evidence indicating that the policy alan argues in favor of would have damaging consequences that outweigh its positive consequences.

(e) pointing out that alan s argument mistakenly identifies something as the cause of a trend when it is really an effect of that trend

13. astronomers have long thought that the irregularity in the orbit of the planet neptune was adequately explained by the gravitational pull exerted on neptune by the planet pluto the most recent observations of pluto, however indicate that this planet is much too small to exert the amount of gravitational pull on neptune that astronomers once thought it did

if the statements above are true, they provide the most support for which one of the following?

(a) neptune is somewhat larger than scientists once believed it to be

(b) the orbit of neptune is considerably more irregular than scientists once thought it was

(c) there exists another as yet undiscovered planet with an orbit beyond that of pluto

(d) the gravitational pull of pluto is not the sole cause of neptune s irregular orbit

(e) further observations of pluto will eventually show it to be even smaller than it is now thought to be

questions 14-15

in most corporations the salaries of executives are set by a group from the corporation s board of directors. since the board s primary mission is to safeguard the economic health of the corporation rather than to make its executives rich, this way of setting executives salaries is expected to prevent excessively large salaries but , clearly this expectation is based on poor reasoning after all, most members of a corporation s board are themselves executives of some corporation and can expect to benefit from setting generous benchmarks for executives salaries.

14. the point made by the author is that the most common way of setting executives salaries might not keep those salaries in bounds because

(a) most corporals exectives, thanks to their generous salaries, are not financially dependent on money earned as board members

(b) most corporals executives might be less generous in setting their own salaries than the board members actually setting them are

(c) many board members might let their self-interest as executives interfere with properly discharging their role as board members in setting executives salaries

(d) many board members who set executives salaries unreasonably high do so because they happen to be on the board of a corporation of which they expect later to become executives

(e) many board members are remunerated generously and wish to protect this source of income by pleasing the executives to whom they owe their appointments on the board

15. which one of the following practices is vulnerable to a line of criticism most parallel to that used in the argument in the passage?

(a) in medical malpractice suits giving physicrans not directly involved in a suit a major role in determining the damages due to successful plaintiffs

(b) in a legislature, allowing the legislators to increase their own salaries only if at least two-thirds of them vote in favor of an increase

(c) to work both fast an accurately by paying them by the piece but counting only pieces of acceptable quality

(d) in a sports competition decided by judges scores selecting the judges from among people retured from that sport after successful careers

(e) in a business organization distributing a group bonus among the members of a task force on the basis of a confidential evaluation by each member of the contribution made by each of the others.

16. consumer advocate one advertisement that is deceptive, and thus morally wrong, states that gram for gram, the refined sugar used in out chocolate pies is no more fattening than the sugars found in fruits and vegetables“ this is like trying to persuade someone that chocolate pies are not fattening by saying that, calorie for calorie they are no more fattening than celery true but it would take a whole shopping cart full of celery to equal a chocolate pie s worth of calories

a dvertiser this advertisement cannot be called deceptive. it is, after all true

which one of the following principles, if established would do most to support the consumer advocate s position against the advertiser s response?

(a) it is morally wrong to seek to persuade by use of deceptive statements

(b) a true statement should be regarded as deceptive only if the person making the statement believes it to be false, and thus intends the people reading or hearing it to acquire a false belief.

(c) to make statements that impart only a small proportion of the information in one s possession should not necessarily be regarded as deceptive

(d) it is morally wrong to make a true statement in a manner that will deceive hearers or readers of the statement into believing that it is false

(e) a true statement should be regarded as deceptive if it is made with the expectation that people hearing or reading the statement will draw a false conclusion from it.

17. members of the amazonian akabe people commonly take an early-morning drink of a tea made from the leaves of a forest plant. although they greatly enjoy this drink, at dawn they drink it only in small amounts. anthropologists hypothesize that since this tea is extraordinarily high in caffeine, the explanation for the akabe s not drinking more of it at dawn is that high caffeine intake would destroy the surefootedness that their daily tasks require.

which one of the following, if true, most seriously calls the anthropologists explanation into question?

(a) the drink is full of nutrients otherwise absent from the akabe diet

(b) the akabe also drink the tea in the evening, after their day s work is done.

(c) the leaves used for the tea contain a soluble narcotic.

(d) akabe children are introduced to the tea in only a very weak form.

(e) when celebrating, the akabe drink the tea in large quantities.

18. all of the cargo ships of the blue star liner are over 100 meters long, and all of its passenger ships are under 100 meters long. most of the ships of the blue star line were built before 1980. all of the passenger and cargo ships of the gold star line were built after 1980, and all are under 100 meters long. the dockside facilities of port tropica, which is open only to ships of these two lines, can accommodate only those ships that are less than 100 meters long. the s.s. coral is a cargo ship that is currently docked at port tropica

if the statements above are true, which one of the following must be true on the basis of them?

(a) the s.s.coral was built after 1980.

(b) the s.s.coral belongs to the blue star line.

(c) port tropica is served only by cargo ships.

(d) port tropica is not served by ships of the blue start line.

(e) all of the ships of the blue star line are older than any of the ships of the gold star line.

19. spectroscopic analysis has revealed the existence of frozen nitrogen, methane, and carbon monoxide on the surface of pluto. such ices have a tendency to vaporize, producing an atmosphere. since the proportion of any gas in such an atmosphere depends directly on how readily the corresponding ice vaporizes, astronomers have concluded that the components of pluto s atmosphere are nitrogen, carbon monoxide, and methane, in order of decreasing abundance.

the astronomers argumetn relies on which one of the following assumptions?

(a) there is no more frozen nitrogen on the surface of pluto than there is either frozen carbon monoxide or methane.

(b) until space probes reach pluto, direct analysis of the atmosphere is impossible.

(c) there is no frozen substance on the surface of pluto that vaporizes more readily than methane but less readily than carbon monoxide.

(d) nitrogen is found in the atmosphere of a planet only if nitrogen ice is found on the surface of that planet.

(e) a mixture of nitrogen, carbon monoxide, and methane is characteristic of the substances from which the solar system formed.

20. ann will either take a leave of absence from technocomp and return in a year or else she will quit her job there; but she would not do either one unless she were offerend a one-year teaching fellowship at a prestigious university. technocomp will allow her to take a leave of absence if it does not find out that she has been offered the fellowship, but not otherwise, therefore, ann will quit her job at tedhnocomp only if technocomp finds out she has been oftered the fellowship.

which one of the following, if assumed, allows the conclusion above to be properly drawn?

(a) technocomp will find out about ann being offered the fellowship only if someone informs on her.

(b) the reason ann wants the fellowship is so she can quit her job at technocomp.

(c) technocomp does not allow any of its employees to take a leave of absence in order to work for one of its competitors.

(d) ann will take a leave of absence if technocomp allows her to take a leave of absence.

(e) ann would be offered the fellowship only if she quit her job at technocomp.

21. if a mechanical aerator is installed in a fish pool, the water in the pool can be properly aerated. so, since john s fish pool does not have a mechanical aerator, it must be that his pool is not properly aerated. without properly aerated water, fish cannot thrive. therefore, any fish in john s fish pool will not thrive.

which one of the following arguments contains an error of reasoning that is also contained in the argument above?

(a) if alum is added to pickle brine, brine can replace the water in the pickles. therefore, since paula does not add alum to her pickle brine, the water in the pickles cannot be replaced by brine. unless their water is replaced with brine, pickles will not stay crisp. thus, paula s pickles will not stay crisp.

(b) if pectin is added to jam, the jam will gel. without a setting agent such as pectin, jam will not ge. so in order to make his jam gel. harry should add a setting agent such as pectin to the jam.

(c) if stored potatoes are not exposed to ethylene the potatoes will not sprout. beets do not release ethylene. therefore, if sara stores her potatoes together with beets, the potatoes will not sprout.

(d) if a carrot patch is covered with mulch in the fall. the carrots can be left in the ground until spring without a mulch cover, carrots stored in the ground can suffer frost damage. thus, since kevin covers his carrot patch with mulch in the fall, the carrots can safely be left in the ground.

(e) if tomatoes are not stored in a dark place, their seeds sometimes sprout. sprouted seeds can make tomatoes inedible. therefore, since maria does not store her tomatoes in a dark place some of maria s tomatoes could be inedible.

questions 22-23

antinuclear activist: the closing of the nuclear power plant is a victory for the antinuclear cause. it also represents a belated acknowledgment by the power industry that they cannot operate such plants safely.

nuclear power plant manager : it represents no such thing. the availability of cheap power from nonnuclear sources. together with the cost of mandated safety inspections and safety repairs, made continued operation uneconomic. thus it was not safety considerations but economic considerations taht dictated the plant s closing.

22. the reasoning in the manager s argument is flawed because the argument

(a) fails to acknowledge that the power industry might now believe nuclear power plants to be unsafe even though this plant was not closed for safety reasons

(b) overlooks the possibility that the soruces from which cheap power is available might themselves be subject to safety concerns

(c) mistakes the issue of what the closure of the plant represents to the publie for the issue of what the managers reason for the closure were

(d) takes as one of its premises a view about the power industry s attitude toward nuclear safety that contradicts the activist s view

(e) counts as purely economic considerations some expenses that arise as a result of the need to take safety precautions

23. which one of the following if true, most strongly supports the activist s claim of victory?

(a) the plant had reached the age at which its operating license expired.

(b) the mandate for inspections and repairs mentioned by the manager was recently enacted as a result of pressure from antinuclear groups.

(c) the plant would not have closed if cheap power from nonnuclear sources had not been available.

(d) per unit of electricity produced the plant had the highest operating costs of any nuclear power plant.

(e) the plant that closed had been able to provide backup power to an electrical network when parts of the network became overloaded.

questions 24-25

statistician changes in the sun s luminosity correlate exceedingly well with average land temperatures on earth. clerly—and contrary to accepted opinion among meteorologists—the sun s lumionsity essentially controls land temperatures on earth.

meteorologist: i disagree any professional meteorologist will tell you that in a system as complicated as that giving rise to the climate, no significant aspect can be controlled by a single variable

24. the rejection by the meteorologist of the statistician s conclusion employs which one of the following techniques of argumentation?

(a) supporting a conclusion about a specific case by invoking a relevant generalization

(b) producint a single counterexample that establishes that a generalization is false as state

(c) reanalyzing a correlation as reflecting the multiple effects of a single cause

(d) rejecting a conclusion because it is a proposition that cannot be experimentally tested

(e) pointing out that potentially unfavorable evident has been systematically neglected

25. the reasoning in the meteorologist s counterargument questionable because that argument

(a) rejects a partial explanation, not because it is incorrect but only because it is not complete

(b) fails to distinguish phenomena that exist independently of a particular system from phenomena that exist only as part of the system.

(c) calls into question the existence of a correlation when the only real issue is that of how to interpret the correlation

(d) dismisses a hypothesis on the grounds that is fail to deal with anymatters of scientific significant

(e) appeals to the authoritativeness of an opinion without evaluating the merit of a putative counterexample

【篇3】LSAT考试全真试题三SECTION2

关于LSAT考试考试全真试题三

LSAT考试考试全真试题三 SECTION 1

SECTION 1

Time-35 minutes

24 Questions

Directions: Each group of questions in this section is based on a set of conditions. In answering some of the questions, it may be useful to araw a rough diagram. Choose the resoonse that most accurately and completely answers each question and blacken the corresponding space on your answer sheet.

Questions 1-6

Seven students-fourth-year students Kim and Lee; third-year students Pat and Robin: and second-year students Sandy, Tety and Val-and only those seven, are being assigned a rooms of equal size in a dormitory. Each room assigned must have either one or two or three students assigned to it and will accordingly be called either a single or a double or a triple. The seven students are assigned to moms in accordence with the following conditions:

Lio fourth-year student can be assigned to a triple.

No second-year student can be assigned to a single.

Lee and Pobin must not share the same room

Kim and Pat must share the same room.

1. Which one of the following is a combination of rooms to which the seven students could be assigned?

(A) two triples and one single

(B) one triple and four singles

(C) three doubles and a stngle

(D) two doubles and three singles

(E) one double and five singles

2. It the room assigned to Robin is a single, which one of the following could be true?

(A) There is exactly one double that has a second-year student assigned to it.

(B) Lee is assigned to a stngle.

(C) Sandy Fat and one other student are zseigned to a triple together.

(D) Lixactly three of the rooms assigned to the students are singles

(E) Exactly two of the rooms assigned to the students are doubles.

3. Which one of the following must be true?

(A) Lee is assigned to a single

(B) Pat sharts a double with another student

(C) Robin shares a double with another student

(D) Two of the second-year students share a double with each other

(E) Neither of the third-year students is assigned to a single

4. If Robin is assigred to a triple, which one of the following must be true?

(A) Lee is assigned to a single

(B) Two second-year students share a double with each other

(C) None of the rooms assigned to the students is a single

(D) Two of the rooms assigned to the students are singles.

(E) Three of the rooms assigned to the students are singles

5. If Terry and Val assigned to different doubles from each other, other, then it must be true of the students rooms that exactly

(A) one is a single

(B) two are singles

(C) two are doubles

(D) one is a triple

(E) two are triples

6. Which one of the following could be true?

(A) The two fourth-year students are assigned to singles.

(B) The two fourth-year students share a double with cach other.

(C) Lee shares a room with a second-year student

(D) Lee shares a room with a third-year student

(E) Pat shares a triple with two other students

Questions 7-11

A worker will colored light bulbs into a billboard equipped with exactly three light sockets, which are labled lights 1, 2, and 3. The worker has three green bulbs, three purple bulbs, and three yellow bulbs. Seiection of bulbs for the sockets is governed by the following conditions:

Whenever light 1 is purple, light 2 must be yellow.

Whenever light 2 is purple, light 1 must be green.

Whenever light 3 is either purple or yellow, light 2 must be purple.

7. Which one of the following could be an accurate list of the colors of light bulbs selected for lights 1, 2 and 3, respectively?

(A) green, green, yellow

(B) purple, green, green

(C) purple, purple, green

(D) yellow, purple, green

(E) yellow, yellow, yellow

8. If light 1 is yellow, then any of the following can be true, EXCEPT:

(A) Light 2 is green.

(B) Light 2 is purple

(C) Light 3 is green

(D) Light 3 is purple

(E) Light 3 is yellow

9. There is exactly one possible color sequence of the three lights if which one of the following is true?

(A) Light 1 is purple.

(B) Light 2 is purple.

(C) Light 2 is yellow

(D) Light 3 is purple.

(E) Light 3 is yellow

10. If no green bulbs are selected, there are exactly how many possible different color sequences of the three lights?

(A) one

(B) two

(C) three

(D) four

(E) five

11. If no two lights are assigned light bulbs that are the same color as each other, then which one of the following could be true?

(A) Light I is green, and light 2 is purple.

(B) Light I is green, and light 2 is yellow.

(C) Light I is purple, and light 2 is yellow.

(D) Light I is yellow, and light 2 is green.

Questions 12-17

An attorney is scheduling interviews with witnesses for a given week. Monday through Saturday. Two full consecutive days of the week must be reserved for interviewing hostile withesses. In addition, nonhostile witnesses Q, R, U, X, Y, and Z will each be interviewed exactly once for a full morning or afternoon. The only witnesses who will be interviewed simultaneously with each other are Q and R. The following conditions apply.

X must be interviewed on Thursday morning

Q must be interviewed at some time before X.

U must be interviewed at some time before R

Z must be interviewed at some time after X and at some time after Y.

12. Which one of the following is a sequence, from first to last, in which the nonhostile witnesses could be interviewed?

(A) Q with R, U, X, Y, Z

(B) Q, U, R, X, with Y, Z

(C) U, X, Q, with R, Y, Z

(D) U, Y, Q, with R, X, Z

(E) X, Q, with U, Z, R, Y

13, Which one of the following is acceptable as a complete schedule of witnesses for Tuesday morning. Tuesday afternoon, and Wednesday morning,respectively?

(A) Q, R, none

(B) R, none, Y

(C) U, Y, none

(D) U, Y, none

(E) Y, Z, none

14.If Y is interviewed at some time after X, which one of the following must be a day reserved for interviewing hostile witnesses?

(A) Monday

(B) Tuesday

(C) Wednesday

(D) Friday

(E) Saturday

15. If R is interviewed at some time after Y which one of the following must be a day reserved for interviewing hostile witnesses?

(A) Monday

(B) Tuesday

(C) Wednesday

(D) Thursday

(E) Friday

16. If on Wednesday afternoon and on Monday the attomey conducts no interviews, which one of the following be true?

(A) Q is interviewed on the same day as U

(B) R is interviewed on the same day as Y

(C) Y is interviewed on the same day as U

(D) Y is interviewed on the same day as Wednesday

(E) Z is interviewed on the same day as Friday

17. If Z is interviewed on Saturday morning which one of the following can be true?

(A) Wednesday is a day reserved for interiewing hostile witnesses.

(B) Friday is a day reserved for interviewing hostile witnesses.

(C) R is interviewed on Thursday

(D) U is interviewed on Tuesday

(E) Y is interviewed at some time before Thursday

Questions 18-24

During a four-week period, cach of seven previously unadvertised products-G, H, J, K, L, M, and O-will be advertised. A different pair of these products will be advertised each week. Exactly one of the products will be a member of two of these four pairs. The following constraints must be observed:

J is not advertised during a given week unless H is advertised during the immediately precceding week.

The product that is advertised during two of the weeks is advertised during week 4 but is not advertised during week 3

G is not advertised during a given week unless either J or else O is also advertised that week.

K is advertised during one of the first two weeks

O is one of the products advertised during week 3

18. Which one of the following could be the schedule of advertisernents?

(A) week 1: G, J; week 2: K, L; week 3: O, M; week 4: H, L

(B) week 1: H, K; week 2: J, G; week 3: O, L; week 4: M, K

(C) week 1: H, K; week 2: J, M; week 3: O, L; week 4: G, M

(D) week 1: H, L; week 2: J, M; week 3: O, G; week 4: K, L

(E) week 1: K, M; week 2: H, J; week 3: O, G; week 4: L, M

19. Which one of the following is a pair of products that CANNOT be advertised during the same week as each other?

(A) H and k

(B) H and M

(C) J and O

(D) K and L

(E) L and M

20. Which one of the following must be advertised during week 2?

(A) G

(B) J

(C) K

(D) L

(E) M

21. Which one of the following CANNOT be the product that is advertised during two of the weeks?

(A) G

(B) H

(C) K

(D) L

(E) M

22. If L is the product that is advertised during two of the weeks, which one of the following is a product that must be advertised during one of the weeks in which L is advertised

(A) G

(B) H

(C) J

(D) K

(E) M

23. Which one of the following is a product that could be advertised in any of the four weeks?

(A) H

(B) J

(C) K

(D) L

(E) O

24. Which one of the following is a pair of products that could be advertised during the same week as each other

(A) G and H

(B) H and J

(C) H and O

(D) K and O

(E) K and O

(F) M and O

【篇4】LSAT考试全真试题三SECTION2

section iii

time—35 minutes

25 questions

directions: the questions in this section are based on the reasoning contained in brief statements or passages. for some questions, more than one of the choices could conceivably answer the question. however, you are to choose the best answer, that is, the response that most accurately and completely answers the question. you should not make assumptions that are by commonsense standards implausible, superfluous, or incompatible with the passage. after you have chosen the best answer, blacken the corresponding space on your answer sheet.

1. everyone sitting in the waiting room of the school s athletic office this morning at nine o clock had just registered for a beginners tennis clinic. john, mary, and teresa were all sitting in the waiting room this morning at nice o clock. no accomplished tennis player would register for a beginners tennis clinic.

if the statements above are true, which one of the following must also be true on the basis of them?

(a) none of the people sitting in the school s athletic office this morning at nine o clock had ever played tennis.

(b) everyone sitting in the school s athletic office this morning at nine o clock registered only for a beginners tennis clinic.

(c) john, mary, and teresa were the only people who registered for a beginners tennis clinic this morning.

(d) john, mary, and teresa were the only people sitting in the waiting room of the school s athletic office this morning at nine o clock

(e) neither john nor teresa is an accomplished tennis player.

2. most people who ride bicycles for pleasure do not ride until the warm weather of spring and summer arrives. yet it is probably more effective to advertise bicycles earlier in the year. most bicycles are purchased in the spring, but once shoppers are ready to shop for a bicycle, they usually have already decided which brand and model of bicycle they will purchase. by then it is generally too late to induce them to change their minds.

the main point of the argument is that

(a) bicycle advertisements are probably more effective if they appear before the arrival of warm spring weather

(b) most bicycle purchasers decide on the brand and model of bicycle that they will buy before beginning to shop for a bicycle

(c) more bicycles are purchassed in the spring than at any other time of year.

(d) in general, once a bicycle purchaser has decided which bicycle he or she intends to purchase, it is difficult to bring about a change in that decision

(e) spring and summer are the time of year in which bicycle riding as a leisure activity is most popular

3. during 1991 the number of people in the town of bayburg who received municipal food assistance doubled, even though the number of people in bayburg whose incomes were low enough to qualify for such assistance remained unchanged.

which one of the following, if true, most helps to resove the apparent discrepancy in the information above?

(a) in 1990 the bayburg town council debated whether or not to alter the eligibility requirements for the food assistance program but ultimately decided not to change them.

(b) in 1990 the bayburg social service department estimated the number of people in bayburg who might be eligible for the food assistance program and then informed the bayburg town council of the total amount of assistance likely to be needed.

(c) during 1991 many residents of a nearby city lost their jobs and moved to bayburg in search of work.

(d) during 1991 the number of applicants for food assistance in bayburg who were rejected on the basis that their incomes were above the maximum allowable limit was approximately the same as it had been in 1990.

(e) during 1991 bayburg s program of rent assistance for low-income tenants advertised widely and then informed all applicants about other assistance programs for which they would be qualified.

4. campaigning for election to provincial or state office frequently requires that a candidate spend much time and energy catering to the interests of national party officials who can help the candidate to win office. the elected officials who campaign for reelection while they are in office thus often fail to serve the interests of their local constituencies.

which one of the following is an assumption made-by the argument?

(a) catering to the interests of national party officials sometimes conflicts with serving the interests of a provincial or state official s local constituencies.

(b) only by catering to the interests of national party officials can those who hold provincial or state office win reelection.

(c) the interests of iocal constituencies are well served only by elected officials who do not cater to the interests of national party officials.

(d) officials elected to provincial or state office are obligated to serve only the interests of constituents who beling to the same party as do the officials.

(e) all elected officials are likely to seek reelection to those offices that are not limited to one term.

5. since professor smythe has been head of the deparment the most distinguished member of the faculty has resigned, fewer new courses have been developed, student has dropped, and the reputation of the department has gone down. these facts provide conclusive evidence that professor smythe was appointed to undermine the department.

the reasoning in the argument is flawed because the argumetn

(a) overlooks the fact that something can have the reputation for being of poor quality without being of poor quality

(b) bases a general claim on a few exceptional instances

(c) assumes that because an action was followed by a change, the action was undertaken to bring about that change.

(d) fails to distinguish between a decline in quantity and a decline in quality

(e) presupposes what it purports to establish

6. books about architectural works. unless they are not intended for a general audience, ought to include discussions of both the utility and the aesthetic appeal of each of the buildings they consider. if they do not, they are flawed. morton s book on italian baroque palaces describes these palaces functional aspects, but fails to mention that the main hall of a palace he discusses at length has a ceiling that is one of the truly breathtaking masterpieces of western art.

if the statements above are true, it would be necessary to establish which one of the following in order to conclude that morton s book is flawed?

(a) morton s deseription of the palaces utility is inaccurate

(b) morton s book does not discuss aspects of the palaces other than utility and aesthetic appeal

(c) morton s book is intended for a general audience.

(d) the passage discussing the palace plays a very important role in helping to establish the overall argument of morton s book.

(e) the palace discussed at length is one of the most aesthetically important of those treated in morton s book.

7. of all the photographs taken of him at his wedding there was one that john and his friends sharply disagreed about. his friends all said that this particular picture did not much resemble him, but john said that on the contrary it was the only photograph that did.

which one of the following, if true about the photograph most helps to explain john s disagreement with his friends?

(a) it, unlike the other photographs of john, showed him in the style of dress he and his friends usually wear rather than the formal clothes he wore at the ceremony.

(b) it was the only photograph taken of john at his wedding for which the photographer had used a flash.

(c) it was a black-and-white photograph, whereas the other photographs that showed john were mostly color photographs.

(d) it was unique in showing john s face reflected in a mirror, the photographer having taken the photograph over john s shoulder.

(e) it was one of only a few taken at the wedding that showed no one but john.

questions 8-9

eva: a “smart highway” system should be installed, one that would monitor areawide traffic patterns and communicate with computers in vehicles or with programmable highway signs to give drivers information about traffic congestion and alternate routes. such a system, we can infer, would result in improved traffic flow in and around cities that would do more than improve drivers tempers; it would decrease the considerable loss of money and productivity that now results from traffic congestion.

lines: there are already traffic reports on the radio. why would a “smart highway” system be any better?

8. eva s argument depends on the assumption that

(a) on “smart highways” there would not be the breakdowns of vehicles that currently cause traffic congestion

(b) traffic lights, if coordinated by the system, would assure a free flow of traffic

(c) traffic flow in and around cities is not now so congested that significant improvement is impossible

(d) the type of equipment used in “smart highway” systems would vary from one city to another

(e) older wehicles could not be fitted with equipment to receive signals sent by a “smart highway” system

9. if eva responded to luis by saying that the current one-minute radio reports are too short to give a sufficient description of overall patterns of traffic congestion, which one of the following, if true, would most strengthen luis s challenge?

(a) bad weather, which radio stations report, would cause traffic to slow down whether or not a “smart highway” system was in operation.

(b) it would be less costly to have radio stations that give continual, lengthier traffic reports than to install a “smart highway” system.

(c) radio reports can take note of congestion once it occurs, but a “smart highway” system could anticipate and forestall it in many instances.

(d) the proposed traffic monitoring would not reduce the privacy of drivers.

(e) toll collection booths, which constiture traffic bottlenecks, would largely be replaced in the “smart highway” system by electronic debiting of commuters accounts while traffic proceeded at full speed.

10. the terms “sex” and “gender” are often used interchangeably. but “sex” more properly refers to biological differences of male and female, while “gender” refers to society s construction of a system that identifies what is masculine and feminine. unlike the set of characteristies defining biological sex, the set of traits that are associated with gender does not sort people into two nonoverlapping groups. the traits characterize people in a complex way, so that a person may have both “masculine” and “feminine” traits.

which one of the following statements best expresses a main point of the argument?

(a) distinctions based on gender are frequently arbitrary.

(b) gender traits are not determined at birth.

(c) masculine gender traits are highly correlated with maleness.

(d) the terms “sex” and “gender” are not properly interchangeable.

(e) society rather than the individual decides what is considered proper behavior.

11. raising the tax rate on essential goods—a traditional means of increasing govemment revenues—invariably turns low-and middle-income taxpayers against the government. hence government officials have proposed adding a new tax on pruchases of luxury items such as yachts, private planes, jewels, and furs. the officials in government revenues while affecting only the wealthy individuals and corporations who can afford to purchase such items.

the answer to which one of the following questions would be most relevant in evaluating the accuracy of the government officials prediction?

(a) will luxury goods be taxed at a higher rate than that at which essential goods are currently taxed?

(b) will be revenues generated by the proposed tax be comparable to those that are currently being generated by taxes on essential goods?

(c) will sales of the luxury items subject to the proposed tax occur at current rates once the proposed tax on luxury items has been passed?

(d) will the proposed tax on luxury items win support for the government in the eyes of low-and middle-income taxpayers?

(e) will purchases of luxury items by corporations account for more of the revenue generated by the proposed tax than will purchases of luxury items by wealthy individuals?

12. in a study of the relationship between aggression and television viewing in nursery school children, many interesting interactions among family styles, aggression, and television viewing were found. high aggression occurred in both high-viewing and low-viewing children and this seemed to be related to parental lifestyle. high-achieving. competitive, middle-class parents, whose children did not watch much television had more aggressive children than parents who planned their lives in an organized, child-centered way, which included larger amounts of television viewing.

which one of the following conclusions is best supported by the passage?

(a) low levels of television viewing often lead to high levels of aggression among children.

(b) the level of aggression of a child cannot be predicted from levels of television viewing alone.

(c) if high-achieving. competitive parents were more child-centered, their children would be less aggressive

(d) high levels of television viewing can explain high levels of aggression among children only when the parents are not child-centered.

(e) parental lifestyle is less important than the amount of television viewing in determining the aggressiveness of children.

13. one of the effects of lead poisoning is an inflammation of the optic nerve, which causes those who have it to see bright haloes around light sources. in order to produce the striking yellow effects in his “sunflowers” paintings, van gogh used naples yellow, a pigment containing lead. since in his later paintings, van gogh painted bright haloes around the stars and sun, it is likely that he was suffering from lead poisoning caused by ingesting the pigments he used.

which one of the following is an assumption on which the argument relies?

(a) in van gogh s later paintings he painted some things as he saw them.

(b) van gogh continued to use paints containing lead after having painted the “sunflowers” paintings,.

(c) van gogh did not have symptoms of lead poisoning aside from seeing bright haloes around light sources.

(d) the paints van gogh used in the “sunflowers” paintings had no toxic ingredients other than lead.

(e) the effects of naples yellow could not have been achieved using other pigments.

questions 14-15

politician: the mandatory jail sentences that became law two years ago for certain crimes have enhanced the integrity of our system of justice, for no longer are there two kinds of justice, the kind dispensed by lenient judges and the kind dispensed by severe ones.

pulic advocate: but with judges stripped of discretionary powers, there can be no leniency even where it would be appropriate. so juries now sometimes acquit a given defendant solely beacuse the jurors feel that the mandatory sentence would be too harsh. those juries, then, do not return an accurate verdict on the defendant s guilt. this is why it is imperative that the legislation instituting mandatory jail sentences be repealed.

14. the public advocate responds to the politician s argument by doing which one of the following?

(a) trying to show that the politician s conclusion merely paraphrases the politician s evidence

(b) claiming that the politician s evidence, properly analyzed, has no bearing on the conclusion the politician derives from it.

(c) arguing that leniency is not a trait of individuals but that, rather, it is a property of certain kinds of decisions.

(d) arguing that an analysis of the consequences of certain legislation undermines the politician s conclusion

(e) charging that the politician exaggerated the severity of a problem in order to justify a sweeping solution

15. which one of the following principles, if valid, provides the politician with the strongest basis for countering the public advocate s argument?

(a) juries should always consider whether the sum of the evidence leaves any reasonable doube concerning the defendant s guilt, and in all cases in which it does, they should acquit the defendant

(b) a system of justice should clearly define what the specific actions are that judges are to perform within the system.

(c) asystem of justice should not require any legal expertise on the part of the people selected to serve on juries.

(d) changes in a system of justice in response to some undesirable feature of the system should be made as soon as possible once that feature has been recognized as undesirable.

(e) changes in a system of justice that produce undesirable consequences should be reversed only if it is not feasible to ameliorate those undesirable consequences through furhter modification.

16. researchers studying artificial sweeteners have long claimed that the perception of sweetness is determined by the activation of a single type of receptor on the tongue, called a sweetness receptor. they have also claimed that any given individual molecule of substance can activate at most one sweetness receptor and that the fewer molecules that are required to activate a receptor, the sweeter that substance will be perceived to be, now the researchers claim to have discovered a substance of which only one molectule is needed to activate any sweetness receptor.

which one of the following conclusions is most strongly supported by the researchers claims, if all of those claims are true?

(a) the more sweetness receptors a person has on his or her tongue, the more likely it is that that person will find sweet sensations pleasurable

(b) in sufficient quantity. the molecules of any substance can activate a sweetness receptor

(c) no substance will be found that is perceived to be sweeter than the substance the researchers have discovered.

(d) a substance that does not activate a sweetness receptor will activate a taste receptor of another type.

(e) the more molecules of a substance that are required to activate a single sweetness receptor. the more bitter that substance will be perceived to be.

17. an editorial in the grandbury daily herald claims that grandburg s voters would generally welcome the defeat of the political party now in control of the grandburg city council. the editorial bases its claim on a recent survey that found that 59 percent of grandburg s registered voters think that the party will definitely be out of power after next year s city council elections.

which one of the following is a principle that, if established, would provide the strongest justification for the editorial s conclusion?

(a) the way voters feel about a political party at a given time can reasonably be considered a reliable indicator of the way they will continue to feel about that party, barring unforeseeable political developments.

(b) the results of surveys that gauge current voter sentiment toward a given political party can legitimately be used as the basis for making claims about the likely future prospects of that political party.

(c) an increase in ill-feeling toward a political party that is in power can reasonably be expected to result in a corresponding increase in support for rival political parties.

(d) the proportion of voters who expect a given political possibility to be realized can legitimately be assumed to approximate the proportion of voters who are in favor of that possibility being realized.

(e) it can reasonably be assumed that registered voters who respond to a survey regarding the outcome of a future election will exercise their right to vote in that election.

18. prolonged exposure to nonionizing radiation—electromagnetic radiation at or below the frequency of visible light—increases a person s chances of developing soft-tissue cancer. electric power lines as well as such electrical appliances as electric blankets and video-display terminals are sources of nonionizing radiation.

which one of the following conclusions is best supported by the statements above?

(a) people with short-term exposure to nonionizing radiation are not at risk of developing soft-tissue cancers.

(b) soft-tissue cancers are more common than other cancers.

(c) soft-tissue cancers are frequently cured spontaneously when sources of nonionizing radiation are removed from the patient s home.

(d) certain electrical devices can pose health risks for their users.

(e) devices producing electromagnetic radiation at frequencies higher than that of visible light do not increase a person s risk of developing soft-tissue cancers.

19. in the first decade following the founding of the british labour party, the number of people regularly voting for labour increased fivefold. the number of committed labour voters increased a further fivefold during the party s second decade. since the increase was thus the same in the first as in the second decade, the often-made claim that the labour party gained move voters in the party s second decade than in its first is clearly false.

the reasoning in the argument is flawed because the argument

(a) fails to specify dates necessary to evaluate the truth of the conclusion, even though the argument depends on distinguishing between two time periods

(b) draws a conclusion that cannot be true if all the data advanced in its support are true

(c) relies on statistical evidence that, strictly speaking, is irrelevant to establishing the conclusion drawn

(d) fails to allow for the possibility that the policy positions advocated by the labour party changed during the period in question

(e) overlooks the possibility that more elections were held in one of the two decades than were held in the other

questions 20-21

a number of seriously interested amateur astronomers have tested the new exodus refractor telescope. with it, they were able to observe in crisp detail planetary features that were seen only as fuzzy images in their 8-inch (approximately 20-centimeter) newtonian telescopes, even though the 8-inch telescopes, with their wider apertures, gather more light than the 4-inch (approximately 10-centimeter) exodus. given these amateur astronomers observational findings, any serious amateur astronomers ought to choose the exodus if she or he is buying a telescope for planetary observation.

20. the argument proceeds by

(a) evaluating the credibility of claims made by a particular group

(b) detailing the ways in which a testing situation approximates the conditions of ordinary use

(c) placing a phenomenon in a wider context in order to explain it

(d) supporting a recommendation to a group on the basis of the experience of a subset of that group

(e) distinguishing between the actual reasons why a certain group did a particular thing and the best reasons for doing that thing.

21. which one of the following most seriously weakens the argument?

(a) telescopes of certain types will not perform well unless they have been precisely collimated a delicate adjustement requriring deftness.

(b) image quality is only one of several different factors that, taken together, should determine the choice of a telescope for planetary observation.

(c) many serious amateur astronomers have no intention of buying a telescope for planetary observation.

(d) the comparisons made by the amateur astronomers were based on observations made during several different observation sessions.

(e) the substance used to make the lenses of exodus telescopes differs from that used in the lenses of other telescopes.

22. anatomical bilateral symmetry is a common trait. it follows, therefore, that it confers survival advantages on organisms. after all, if bilateral symmetry did not confer such advantages, it would not be common.

the pattern of reasoning in which one of the following arguments is most similar to that in the argument above?

(a) since it is sawyer who is negotiating for the city government, it must be true that the city takes the matter seriously. after all, if sawyer had not been available, the city would have insisted that the negotiations be deferred.

(b) clearly, no candidate is better qualified for the job than trumbull. in fact, even to suggest that there might be a more highly qualified candidate seems absurd to those who have seen trumbull at work.

(c) if powell lacked superior negotiating skills, she would not have been appointed arbitrator in this case. as everyone knows, she is the appointed arbitrator, so her negotiating skills are detractors notwithstanding bound to be superior.

(d) since varga was away on vacation at the time, it must have been rivers who conducted the secret negotiations. any other scenario makes little sense, for rivers never does the negotiating unless varga is unavailable.

(e) if wong is appointed arbitrator, a decision will be reached promptly. since it would be absurd to appoint anyone other than wong as arbitrator, a prompt decision can reasonably be expected.

23. electrical engineers have repeatedly demonstrated that the best solid-state amplifiers are indistinguishable from the best vacuum-tube amplifiers with respect to the characteristies commonly measured in evaluating the quality of an amplifier s musical reproduction. therefore, those music lovers who insist that recorded music sounds better when played with the best vacuumtube amplifier must be imagining the difference in quality that they claim to hear.

which one of the following, if true, most seriously weakens the argument?

(a) many people cannot tell from listening to it whether a recording is being played with a very good solid-state amplifier or a very good vacuum-tube amplifier.

(b) the range of variation with respect to the quality of musical reproduction is greater for vacuumtube amplifiers than for solid-state amplifiers.

(c) some of the characteristies that are important in determining how music sounds to a listener cannot be measured.

(d) solid-state amplifiers are more compact, use less power, and generate less heat than vacuum-tube amplifiers that produce a comparable volume of sound.

(e) some vacuum-tube amplifiers are clearly superior to some solid-state amplifiers with respect to the laboratory to evaluate the quality of an amplifier s musical reproduction.

24. explanation must be distinguished from justification every human action potentially has an explanation that is with sufficient knowledge it would be possible to give an accurate description of the causes of that action. an action is justified only when the person performing the action has sufficient reasons for the action. according to many psychologists, even when there is a justification for an action, that justification often forms no part of the explanation. the general principle, however, is that only an action whose justification, that is, the reasons for the action, forms an essential part of its explanation is rational.

if the statements in the passage are correct, which one of the following can be properly concluded form them?

(a) when a human action is justified, that action has no explanation.

(b) if there are any reasons among the causes of an action, then that action rational

(c) some psychologists believe that the justification for an action never forms an essential part of its explanation

(d) there are actions whose causes cannot be discovered

(e) if any human actions are rational then reasons must sometimes be causes of actions

25. at the company picnic all of the employees who participated in more than four of the scheduled events, and only those employees were eligible for the raffle held at the end of the day. since only a small proportion of the employees were eligible for the raffle, most of the employees must have participated in fewer than four of the scheduled events.

which one of the following arguments exhibits a flawed pattern of reasoning most like that exhibited by the argument above?

(a) only third-and fourth-year students are allowed to keep cars on campus. since one quarter of the third-year students keep cars on campus and one half of the fourth-year students keep cars on campus, it must be that fewer third-year students than fourth-year students keep cars on campus.

(b) only those violin students who attended extra rehearsal sessions were eligible for selection as solists. since two of the violin students were selected as soloists, those two must have been the only violin students who attended the extra sessions

(c) the only students honored at a special banquet were the band members who made the dean s list last semester. since most the band members were honored, most of the band members must have made the dean s list.

(d) all of the members of the service club who volunteered at the hospital last summer were biology majors. since ten of the club members are biology majors, those ten members must have volunteered at the hospital last summer

(e) all of the swim team members who had decreased their racing times during the season were given awards that no other members were given. since fewer than half the team members were given such awards, the racing times of more than half the team members must have increased during the season.

【篇5】LSAT考试全真试题三SECTION2

section ii

time—35 minutes

25 questions

directions: the questions in this section are based on the reasoning contained in brief statements or passages. for some questions, more than one of the choices could conceivably answer the question. however, you are to choose the best answer, that is the response that most accurately and completely answers the question. you should pot make assumptions that are by commonsense standards implausible, superfluous, or incompatible with the passage after you have chosen the best answer, blacken the corresponding space on you answer sheet.

1. press release a comprehensive review evaluating the medical studies done up to the present time has found no reason to think that drinking coffee in normal amounts harms the coffee-drinker s heart so coffee drinkers can relax and enjoy their beverage—it is safe to drink coffee

which one of the following points to a weakness in the reasoning in the press release s argument?

(a) the review was only an evaluation of studies and did not itself undertake to study patients.

(b) the health of the heart is not identical with the general health of the body

(c) coffee drinkers might choose to eat along with their coffee foods contaming substances that harm the heart

(d) other beverages besides coffee might contain stimulants that have some effect on the heart

(e) drinking unusually large amounts of coffee could be caused by stress that itself directly harms the heart

2. all people prefer colors that they can distinguish easily to colors that they have difficulty distinguishing. infants can easily distinguish bright colors but, unlike adults, have difficulty distinguishing subtle shades. a brightly colored toy for infants sells better than the same toy in subtle shades at the same price

which one of the following conclusions is most strongly supported by the information in the passage?

(a) infants prefer bright primary colors to bright secondary colors

(b) color is the most important factor in determining which toys an infant will prefer to play with

(c) individual infants do now have strong preferences for one particular bright color over other bright colors

(d) the sales of toys ofr infants reflect the preferences of infants in at least one respect

(e) toy makers study infants to determine what colors the infants can distinguish easily

3. a group of unusual meteorites was found in shergotty. india. their structure indicates that they originated on one of the geologically active planets. mercury, venus, or mars because of mercury s proximity to the sun any material dislodged from that planet s surface would have been captured by the sun, rather than falling to earth as meteorites, nor could venus be the source of the meteorites, because its gravity would have prevented dislodged material from escaping into space the meteorites, therefore, probably fell to earth after being dislodged from mars, perhaps as the result of a collision with a large object

the argument derives its conclusion by

(a) offering a counterexample to a theory

(b) eliminating competing alternative explanations

(c) contrasting present circumstances with past circumstances

(d) questioning an assumption

(e) abstracting a general principle from specific data

4. because quitting smoking is very stressful and leads to weight gain, it is difficult to do. the key to quitting however, may be as simple as replacing an unhealthy activity with a healthy one in one study half of htose attempting to quit were assigned to a smoking-cessation program alone, and the other half were assigned to the same program plus fifteen weeks of aerobic exercise the one-month mark none in the first group had quit but 40 percent of those in the second group had not smoked

each of the following, if true, provides some support for the argument except:

(a) regular exercise prevents weight gain

(b) each group in the study included four hundred randomly selected participants

(c) nonsmokers accustomed to regular exercise do not gain weight when they stop exercising

(d) aerobic exercise can stimulate the brain s production of endorphins. which reduce tension

(e) of those in the second group in the study 38 percent had not smoked at the one-year mark.

5. altogethe, the students in ms. tarnowski s milton elementary school class collected more aluminum cans than did the students in any of the school s other classes therefore, the milton student who collected the most aluminum cans was in ms tarnowski s class

which one of the following arguments contains flawed reasoning that is most paralled to that in the argument above?

(a) altogether, more trees were planted by the students in mr kelly s class than were planted by those in mr liang s class and mr jackson s class combined therefore. mr kelly s students planted more trees than mr jackson s students planted

(b) more than half of milton elementary school s students play in the band and more than half of the school s students sing in the choir therefore, every student at milton elementary school either plays in the band or sings in the choir

(c) mr rowe s milton elementary school class raised more money by selling candy bars than ms hunt s class raised by holding a raffle. therefore, the number of candy bars sold by mr rowe s class was greater than the number of raffle tickets sold by ms. hunt s class

(d) the total number of tickets to the school fair sold by the students in ms. ramirez s milton elementary school class was greater than the number sold by milton students from any other class. therefore, the milton student who sold the most tickets to the school fair was a student in ms rairez s class

(e) ms. ventura s milton elementary school class assembled more birdhouses than did any of the school s other classes. since ms ventura s class had fewer students than any other milton class, her students assembled more birdhouse on average than did the students in any other milton class

6. several excellent candidates have been proposed for the presidency of united wire and each candidate would bring to the job different and experience if the others are compared with jones however it will be apparent that none of them has her unique set of qualifications jones therefore is best qualified to be the new president of united wire

the argument is vulnerable to criticism on the ground that it

(a) uses flattery to win over those who hold an opposing position

(b) refutes a distorted version of an opposing position

(c) seeks to distinguish one member of a group on the basis of something that applies to all

(d) supports universal claim on the basis of a single example

(e) describes an individual in terms that appropriately refer only to the group as a whole

7. a neighborhood groupp plans to protest the closing of the neighborhood s only recreation center on the grounds that to do so would leave the neighborhood without local access to a recreation center ”our neighborhood already has the most residents per center of any neighborhood in the city“ complained one resident, ”and closing this center would make the situation unacceptable since access to recreational facilities is a necessity for this neighborhood“

each of the following if true weakens the resident s argument except

(a) a large number of the neighborhood s residents are unable to travel outside their locality to gain access to recreational facilities

(b) children, the main users of recreational facilities make up a disproportionately small segment of the neighborhood s population

(c) often the recreation center in the neighborhood is open but not being used.

(d) programs that are routinely filled at other recreation centers must be canceled at the ngighborhood s recreation center due to lack of interest

(e) as people become more involved in computers and computer games recreation centers are becoming increasingly less important

8. sociologist: the claim that there is a large number of violent crimes in our society is false, for this claim is based upon the large number of stories in newspapers about violent crimes. but since violent crimes are very rare occurrences, newspapers are likely to print stories about them.

the sociologist s argument is flawed because it

(a) presupposes that most newspaper stories are about violent crime

(b) presupposes the truth of the conclusion it is attempting to establish

(c) assumes without warrant that the newspaper stories in question are not biased

(d) mistakes property of each member of a group taken as an individual for a property of the group taken as a whole

(e) uncritically draws an inference from what has been true in the past to what will be true in the future

9. historian anyone who thinks that the terrors of the ancient rgeime of q were exclusively the work of fanatics is overlooking a basic truth the regime was made up primarily of ordinary people enthusiasically seeking paradist. the regime executed many people in pursuit of its goal. but it later became clear that paradise as they defined it, is unrealizable so at least some of the ordinary people of q were in fact murdreers

which one of the following principles, if valid, provides the most support for the historian s argumentation?

(a) the pursuit of paradise does not justify murder

(b) the pursuit of paradise justifies fanaticism

(c) execution in pursuit of what is later found to be unattainable constitutes murder

(d) fanaticism in pursuit of paradise constitutes inhumanity

(e) enthusiasm in pursuit of what is eventually found to be unattainable constitutes fanaticism

10. economist: the economy seems to be heading out of recession. recent figures show that consumers are buying more durable goods than before indicating that they expect economic growth in the near future

that consumers are buying more durable goods than before figures in the economist s argument in which one of the following ways?

(a) it is the phenomenon that the argument seeks to explain

(b) its truth is required in order for the argument s conclusion to be true

(c) it is an inference drawn from the premise that the recession seems to be ending

(d) it is an inference drawn from the premise that consumers expect economic growth in the near future

(e) it is the primary evidence from which the argument s conclusion is drawn

11. not surprisingly, there are no professors under the age of eighteen and as is well known no one under eighteen can vote legally. finally some brilliant people are professors some are legal voters and some are under eighteen

if the statements above are true, then on the basis of them which one of the following must also be true?

(a) no professors are eighteen-year-olds

(b) all brilliant people are either professors legal voters or under eighteen

(c) some legal voters are not professors

(d) some professors are neither legal voters not brilliant people

(e) some brilliant people are neither professors nor legal voters

12. for years scientists have been scanning the skies in the hope of finding life on other planets. but in spite of the ever-increasing sophistication of the equipment they employ, some of it costing hundreds of millions of dollars not the first shred of evidence of such life has been forthcoming and there is no reason to think that these scientists will be any more successful in the future no matter how much money is invested in the search the dream of finding extraterrestrial life is destined to remain a dream as science s experience up to this point should indicate

which one of the following most accurately states the main point of the argument?

(a) there is no reason to believe that life exists on other planets

(b) the equipment that scientists employ is not as sophisticated as it should be

(c) scientists searching for extraterrestrial life will not find it

(d) only if scientists had already found evidence of life on other planest would continued search be justified

(e) we should not spend money on sophisticated equipment to aid in the search for extraterrestrial life

13. carl s coffee emporium stocks only two decaffeinated coffees: french roast and mocha java yusef only serves decaffeinated coffee and the coffee he served after dineer last night was smooth and mellow have been french roast so if yusef still gets all his coffee from carl s what he served last night was mocha java

the argument above is most similar in its logical structure to which one of the following?

(a) samuel wants to take three friends to the beach his mother wons both a sedan and a convertible the convertible holds four people so although the sedan has a more powerful engine, if samuel borrows a vehicle from his mother he will borrow the convertible

(b) if anna wants to walk from her house to the office where she works she must either go through the park or take the overpass across the railroad tracks the park paths are muddy and anna does not like using the overpass so the never walks to work

(c) rose can either take a two-week vaction the trail she had planned to hike requires three weeks to complete but is closed by october so if rose takes a vacation it will not be the one she had planned

(d) werdix, inc has offered arno a choice between a job in sales and a job in research arno would like to work at werdix but he would never take a job in sales when another job is available so if he accepts on of these jobs it will be the one in research

(e) if teresa does not fire her assistant her staff will rebel and her department s efficiency will decline losing her assistant would also reduce its efficiency so if no alternative solution can be found theresa s department will become less efficient

14. steven the allowable blood alcohol level for drivers should be cut in half with this reduced limit, social drinkers will be deterred from drinking and driving, resulting in significantly increased highway safety

miguel: no lowering the current allowable blood alcohol level would have little effect on highway statey because it would not address the most important aspect of the drunken driving problem which is the danger to the public posed by heavy drinkers who often drive with a blood alcohol level of twice the current legal limit.

steven and miguel s statements provide the most support for that they would disagree about the truth of which one of the following statements?

(a) social drinkers who drink and drive pose a substantial threat to the public

(b) there is a direct correlation between a driver s blood alcohol level and the driver s ability to drive safely

(c) a driver with a blood alcohol level above the current legal limit poses a substantial danger to the public

(d) some drivers whose blood alcohol level is lower than the current legal limit pose a danger to the public

(e) a driver with a blood alcohol level slightly greater than half the current legal limit poses no danger to the public

questions 15-16

the authors of a recent article examined warnings of an impending wave of extinctions of animal species within the next 100 years. these authors say that no evidence exists to support the idea that the rate of extinction of animal species is now accelerating. they are wrong however consider only the data on fishes 40 species and subspecies of north american fishes have vanished in the twentieth century, 13 between 1900 and 1950, and 27 since 1950

15. which one of the following is the main point of the argument?

(a) there is evidence that the rate of extinction of animal species is accelerating

(b) the future rate of extinction of animal species cannot be determined from available evidence

(c) the rate of extinction of north american fishes is parallel to the rate of extinction of all animal species taken together

(d) forty species and subspecies of north american fishes have vanished in the twentieth century

(e) a substantial number of fish species are in danger of imminent extinction

16. the answer to which one of the following questions would contribute most to an evaluation of the argument?

(a) were the fish species and subspecies that became extinct unrepresenatative of animal species in general with regard to their pattern of extinction?

(b) how numerous were the populations in 1950 of the species and subspecies of north american fishes that have become extinct since 1950?

(c) did any of the species or subspecies of north american fishes that became extinct in the twentieth century originate in regions outside of north america?

(d) what proportion of north american fish species and subspecies whose populations were endangered in 1950 are now thriving?

(e) were any of the species or subspecies of north american fishes that became extinct in the twentiethe century commercially important?

17. after the second world war, the charter of the newly formed united nations established an eleven-member security council and charged it with taking collective action in response to threats to world peace. the charter further provided that the five nations that were then the major powers would permanently have sole authority to cast vetoes. the reason given for this arrangement was that the burden of maintaining world peace would rest on the world s major powes and should be required to assume the burden of enforcing a decision it found repugnant

the reasoning given for the structure of the security council assumes that

(a) it does not make sense to provide for democracy among nations when nations themselves are not all democracies

(b) no nation that was not among the major powers at the end of the second world war would become a major power

(c) nations would not eventually gravitate into large geographical bloes, each containing minor powers as well as at least one major power

(d) minor powers would not ally themselves with major powers to gain the prection of the veto exercised by major powers

(e) decisions reached by a majority of nations in response to threats to world peace would be biased in favor of one or more major powers

18. environmental scientist: it is true that over the past ten years, there has been a sixfold increase in government funding for the preservation of wetlands while the total area of wetlands needing such preservation has increased only twofold (although this area was already large ten years ago) even when inflation is taken into account, the amount of funding now is at least three times what it was ten years ago. nevertheless the current amount of government funding for the preservation of wetlands is inadequate and should be augmented

which one of the following, if true most helps to reconcile the environmental scientist s conclusion with the evidence cited above?

(a) the governmental agency responsible for administering wetland-preservation funds has been consistently mismanaged and run inefficiently over the past ten years

(b) over the past ten years, the salaries of scientists employed by the government to work on the preservation of wetlands have increased at a rate higher than the inflation rate

(c) research over the past ten years has enabled scientists today to identify wetlands in need of preservation well before the areas are at serious risk of destruction

(d) more people today scientists and nonscientists alike, are working to preserve all natural resources including wetlands

(e) unlike today funding for the preservation of wetlands was almost nonexistent ten years ago.

19. in australia the population that is of driving age has grown large over the last five years, but the annual number of traffic fatalities has declined. this leads to the conclusion that, overall, the driving-age population of australi consists of more skillful drivers now than five years ago.

each of the statements below, if true, weakens the argument except:

(a) three years ago, a mandatory seat-belt law went into effect throughout australia.

(b) five years ago. australia began a major road repair project

(c) because of increases in the price of fuel australians on average drive less each year than in the preceding year.

(d) the number of hospital emergency facilities in australia has doubled in the last five years

(e) in response to an increase in traffic fatalities. australia instituted a program of mandatory driver education five years ago.

20. anthropological studies indicate that distinct cultures differs in their moral codes. thus, as long as there are distinct cultures there are no values shared across cultures

each of the following, if true, would weaken the argument except”

(a) anthropologists rely on inadequate translation techniques to investigate the values of cultures that use languages different from the anthropologists languages.

(b) as a result of advancing technology and global communication we will someday all share the same sulture and the same values

(c) although specific moral values differ across cultures, more general moral principles, such as “friendship is good” are common to all cultures

(d) the anthropologists who have studied various cultures have been biased in favor of finding differences rather than similarities between distinct cultures

(e) what appear to be differences in values between distinct cultures are nothing more than differences in beliefs about how to live in accordance with shared values.

21. newspaper editor. law enforcenment experts, as well as most citizens, have finally come to recognize that legal prohibitions against gambling all share a common flaw no matter how diligent the effort, the laws are impossible to enforce. ethical qualms notwithstanding, when a law fails to be effective it should not be a law. that is why there should be no legal prohibition against gambling.

which one of the following if assumed. allows the argument s conclusion to be properly drawn?

(a) no effective law is unenforceable

(b) all enforceable laws are effective

(c) no legal prohibitions against gambling are enforceable

(d) most citizens must agree with a law for the law to be effective

(e) most citizens must agree with a law for the law to be enforceable.

22. copernicus s astronomical system is superior to ptolemy s and was so at the time it was proposed, even though at that time all observational evidence was equally consistent with both theories. ptolemy believed that the stars revolved around the earth at great speeds. this struck copernicus as unlikely, he correctly thought that a simpler theory is that the earth rotates on its axis.

the argument most closely conforms to which one of the following principles?

(a) simplicity should be the sole deciding factor in choosing among competing scientific theories

(b) if one theory is likely to be true, and another competing theory is likely to be false, then the one likely to be true is the superior of the two.

(c) if all observational evidence is consistent with two competing theories, the one that is more intuitively true is the more practical theory to adopt.

(d) other things being equal the more complex of two competing theories is the inferior theory

(e) other things being equal, the simpler of two competing theories is the more scientifically important theory.

23. easayist the existence of a moral order in the universe—i.e..an order in which bad is always eventually punished and good rewarded—depends upon human souls being immortal. in some cultures this moral order is regarded as the result of a karma that controls how one is reincarnated, in others it results from the actions of a supreme being who metes out justice to people after their death. but however a moral order is represented if human souls are immortal then if follows that the bad will be punished

which one of the following most accurately describes a flaw in the essayist s reasoning?

(a) from the assertion that something is necessary to a moral order the argument concludes that that thing is sufficient for an element of the moral order to be realized

(b) the argument takes mere beliefs to be established facts

(c) from the claim that the immortality of human souls implies that there is a moral order in the universe the argument concludes that there being a moral order in the universe implies that human souls are immortal

(d) the argument treats two fundamentally different conceptions of a moral order as essentially the same

(e) the argument s conclusion is presupposed in the definition it gives of a moral order.

24. no mathematical proposition can be proven true by observation. it follows that it is impossible to know any mathematical proposition to be true

the conclusion follows logically if which one of the following is assumed?

(a) only propositions that can be proven true can be known to be true

(b) observation alone cannot be used to prove the truth of any proposition

(c) if a proposition can be proven true by observation then it can be known to be true.

(d) knowing a proposition to be true is impossible only if it cannot be prove true by observation

(e) knowing a proposition to be true requires proving it true by observation

25. the publisher of a best-selling self-help book had, in some promotional material, claimed that it showed readers how to become exceptionally successful. of course everyone knows that no book can deliver to the many what by definition, must remain limited to the few exceptional success. thus although it is clear that the publisher knowingly made a false claim. doing so should not be considered unethical in this case

which one of the following principles if valid most strongly supports the reasoning above?

(a) knowingly making a false claim is unethical only if it is reasonable for people to accept the claim as true

(b) knowingly making a false claim is unethical if those making it derive a gain at the expense of those acting as if the claim were true.

(c) knowingly making a false claim is unethical in only those cases in which those who accept the claim as true suffer a hardship greater than the gain they were anticipating

(d) knowingly making a false claim is unethical only if there is a possibility that someone will act as if the claim might be true

(e) knowingly making a false claim is unethical in at least those cases in which for someone else to discover that the claim is false that person must have acted as if the claim were true

【篇6】LSAT考试全真试题三SECTION2

section ii

time—35 minutes

25 questions

questions: the questions in this section are based on the reasoning contained in brief statements or passages. for some questions more than one of the choices could conceivably answer the question. however, you are to choose the best answer that is the response that most accurately and completely answers the question. you should not make assumptions that are by commonsense standards implausible, superfluous, or incompatible with the passage, after you have chosen the best answer, blacken the corresponding space on your answer sheet.

1. the basic ingredients from which cement is made are both cheap and plentiful. materials as common as limestone and clay will do. nevertheless. the price of cement is influenced by the price of oil, because turning the basic ingredients into cement in high-temmerature kilns use large amounts of energy.

which one of the following can be logically inferred from the passage?

(a) oil is one of the basic ingredients that make up cement

(b) oil is a source of energy for some of the kilns used in the making of cement

(c) the higher the price of cement rises, the higher the price of clay rises

(d) whenever oil prices rise cement prices drop

(e) a given amount of cement costs no more than the total cost of its basic ingredients

2. many people do not understand themselves, nor do they try to gain self-understanding these people might try to understand others, but these attempts are sure to fail, because without self-understanding it is impossible to understand others. it is clear from this that anyone who lacks self-understanding will be incapable of understanding others.

the reasoning in the argument is flawed because the argument

(a) mistakes something that is necessary to bring about a situation for something that in itself is enough to bring about that situation

(b) fails to take into account the possibility that not everyone wants to gain a thorough understanding of himself or herself

(c) blames people for something for which they cannot legitimately be held responsible

(d) makes use of the inherently vague term “self-understanding” without defining that term

(e) draws a conclusion that simply restates a claim given in support of that conclusion

questions 3-4

wife: the work of the artist who painted the portrait of my grandparents 50 years ago has become quite popular lately, so the portrait has recently become valuable. but since these sorts of artistic fads fade rapidly, the practical thing to do would be to sell the portrait while it is still worth something, and thereby enable our daughter to attend the college she has chosen.

husband: how could you make such a suggestion? that painting is the only thing you own that belonged to your grandparents. i don t think it s a very good painting, but it has great sentimental value. besides, you owe it to our daughter to keep it in the family as a link to her family s past

3. which one of the following principles, if established, does most to justify the husband s reply?

(a) gifts offered as sentimental tokens of affection should not be accepted if the recipient intends to sell them later for profit

(b) a beautiful work of art is more valuable than the money it could be sold for, whatever the amount

(c) it is more important for parents to provide their children with tangible links to the family s past than it is to enable them to attend the college of their choice.

(d) children and grandchildren have a duty to preserve family heirlooms only if they have promised their parents or grandparents that they would do so.

(e) providing one s children with an education is more important than providing them with material goods, even if the goods have sentimental value.

4. the husband uses which one of the following argumentative techniques in replying to the wife s suggestion?

(a) taking issue with the practicality of her suggestion

(b) questioning her aesthetie judgment

(c) claiming that the reasons she gives are based on emotions rather than on rational considerations

(d) asserting that the evidence she cites in support of her suggestion is false

(e) invoking a competing obligation that he judges to override her practical considerations

5. questions have arisen regarding the accuracy of the reports the university s archaeological museum issues on its sales and acquisitions for the year. to forestall controversy, this year s report is being reviewed by three archaeologists from other universities. since these archaeologists will be given full access to all documents on which the report is based, they will be able to determine whether it is indeed accurate.

the reasoning in the argument is flawed because the argument

(a) does not specify whether the reviewers will have access to data about objects that have been in the museum s collection for many years

(b) provides no information regarding the size or quality of the archaeological museum s collection

(c) omits any mention of whether the museum s collection is on display or is available only to researchers

(d) omits any mention of whether the museum s collection is on display or is available only to researchers

(e) does not describe what will occur if the reviewers discover discrepancies between the report and the documents on which it was based

6. engineer: some people argue that the world s energy problems could be solved by mining the moon for helium-3, which could be used for fuel in fusion reactors. but this is nonsense. even if it were possible to mine the moon for helium-3, the technology needed to build viable fusion reactors that could use such fuel is at least 50 years away. if the world s energy problems are not solved before then, it will be too late to solve those problems.

the main point of the argument is that

(a) mining the moon for helium-3 is currently not feasible

(b) fusion reactors that are now being planned are not designed to use hilium-3 as fuel

(c) people who advocate mining the moon for helium-3 do not realize that fusion reactors could be designed to use fuels other than helium-3

(d) mining the moon for helium-3 is not a possible solution to the world s energy problems

(e) if the world s energy problems are not solved within the next 50 years, it will be too late to solve those problems.

LSAT考试全真试题三SECTION2精选6篇

http://m.scabjd.com/yingyu/255707/

相关推荐

猜你喜欢

大家正在看